If two angles are vertical, the sum of their measures is equal to the measure of a right angle true or false?

Answers

Answer 1

Step-by-step explanation:

False,

Vertical angles isn't always complementary l

Answer 2

Answer:

true

Step-by-step explanation:

If two angles are congruent, their measures are equal.


Related Questions

Solve the following system of equations algebraically:
x2 + y2 = 20
2y = x

Answers

Answer:

x = 5

Step-by-step explanation:

x2 + y2 = 20

xy = 20 ÷ 2 ÷2

= 10 ÷ 2

x = 5

i dont know whether this is right

but hope it helps n it works

at which approximate rate in feet per minute did the plane climb to the cruising altitude of 35,000 feet

Answers

17.5 because 35,000 divided by 2000

Please help me I am in need of help!

Answers

Answer:

answer is currents hope this helps

Step-by-step explanation:

The answer is currents hope this helped have a nice day!:)

please help i d don't know i would really appreciate your help

Answers

Answer:

45

Step-by-step explanation:

hello there!

Remember all of the angles in a triangle will add up to equal 180 so to find the missing angle we subtract the given angles from 180

180 - 90 - 45 = 45 so the missing angle would equal 45

(05.06)Which net matches the solid figure shown below?
8 cm
3 cm
6 cm

Answers

Answer:

Can you upload a picture?

Step-by-step explanation:

what’s the missing side length help please

Answers

Answer:

10

Step-by-step explanation:

[tex]a^2 + 576 = 676[/tex]

subtract 676 by 576

=100

[tex]\sqrt{100}[/tex] = 10

Answer:

i think its 7 im not sure

Step-by-step explanation:

Nara wants to determine how much ice it will take to fill her cooler that is in the shape of a rectangular prism. If the cooler has a length of 22 inches, a width of 12 inches, and a height of 1012 inches, how much ice will her cooler hold?

Answers

Answer:

69344 is the area but I don't know how large the ice is so I can not tell you how much ice. Is there any more to the question?

Step-by-step explanation:

So sorry but the volume is V≈2.67×105

Marco can finish a jigsaw puzzle in 3 hours. Working together with Cliff, it takes them 1 hour. How
long would it have taken Cliff working alone?

Answers

Macro can finish the puzzle in 3 hours. We can say that her speed for working is 1/3 or 1/3 puzzle per hour.

Now if they work together, this turns into 1 puzzle per hour.

That must mean that Cliff would be doing the other 2/3 of the puzzle while Macro does his 1/3 in that hour. If Cliff has a speed of doing them 2/3 per hour. Then it would take 1 and a half hour to finish 1 whole puzzle.

Mr.Chamberlain is planning a field trip for his math classes to visit a museum.It costs $300 to rent the bus, then $7.50 per student for entry to the museum Mr. Chamberlain only has a budget of $1,000 from the school. Which equality below represents the number of students he can bring on the trip?

Answers

Answer:

Equation - $1000=$300+$7.50x

Answer - Mr. Chamberlain can included 93 Student Entries

Step-by-step explanation:

$1000 (budget)

$300 (cost to rent bus)

$7.50 (cost of each student entry)

$1000-$300= $700

Now we have found out that Mr. Chamberlain has $700 left to spend on student entries.

$700/$7.50 = 93.333

Round 93.333 down to 93 because you can't have 1/3 of a student (obviously)

93 = the max number of student entries that Mr. Chamberlain can have for the museum field trip

I hope this helps :)

A park is rectangular with a length of 23 miles. If the area of the park is 39 square miles, what is its width? Input your answer as a fraction.

Answers

ANSWER:
17/10

STEP BY STEP:
do 39/23
you get a long decimal
simplify to get 1.7
witch as a fraction is 17/10

hope this helps

what is 9 (4x^2 - 3) simplified?

Answers

Answer:

144x - 27

Step-by-step explanation:

9 (4x² - 3)

9 (16x - 3)

144x - 27

the two the numbers -2.6 and -2.1 are shown on the number line what is the distance between the two points​

Answers

Answer:

0.5

Step-by-step explanation:

The distance between the points will be the difference between the points as shown;

Difference = -2.1 - (-2.6)

Difference = -2.1 + 2.6

Difference = 2.6 - 2.1

Difference = 0.5

Hence the difference between the numbers -2.6 and -2.1 on the number line is 0.5

Can someone help me? I’ll reward points + brainalist

Answers

Answer:

C

it's the set that will add to 180°

36° + 42° + 102° = 180°

Answer:

C. 36°, 42° and 102°

Step-by-step explanation:

This is because here,

36° + 42° + 102° = 180°

Hence, a triangle can be made.

What is the 6th term of (y-3)^8

Answers

This answer

Sorry if wrong :)

Kebetulan w baru beli stok pulpen :v

Abaikan Abaikan saja

Ganteng Doank ( isi sendiri )

Use the original price and the markdown to find the retail price. Original price: $30; Markdown:24%

Answers

Answer:

$22.80

Step-by-step explanation:

which is not a common multiple of 4 and 9

Answers

5 I think.

Step-by-step explanation:


1. If tn =3n - 2, the value of S5 is:
(a) 35
(b) 40
(c) 45
(d) 30
If tn = 2n? + 3n, the value of Sais:
(a) 90
(b) 70
(C) 80
(d) None of these​

Answers

Answer:

1) if tₙ = 3*n - 2, we want to find the value of S(5), which is the sum of the first 5 terms.

We can simply compute the sum directly, it is:

S(5) = t₁ + t₂ + t₃ + t₄ + t₅

S(5) =  (3*1 - 2) +  (3*2 - 2) + (3*3 - 2) + (3*4 - 2) + (3*5 - 2)

S(5) = 35

Then the correct option here is a).

2) This is not correctly written.

I will assume that we the n-th term is:

tₙ = n^2 + 3*n

And we want  S(5) again

Then:

S(5) = t₁ + t₂ + t₃ + t₄ + t₅

S(5) = (1^2 + 3*1) + (2^2 + 3*2) + (3^2 + 3*3) + (4^2 + 3*4) + (5^2 + 3*5)

S(5) = 100

So the correct option is (d)

(Notice that if we had chosen any other S(n), the answer is still option (d) )

If you leave $2300 in an account earning 3% interest, compounded daily, how much money will be in the account after 5 years? (Round your answer to two decimal places.)

Answers

Answer:

$2672.20

Step-by-step explanation:

2300(1+(0.03)/(365))^(5*365)

=$2672.20

A function is shown below.

g(x)=19.60+1.74x

What is the value of g(30)?

Answers

Answer:

g(30) = 71.8

Step-by-step explanation:

g(30) = 19.6 + 1.74(30)

The value of the g(30) is 71.8 if the function is g(x) = 19.60 + 1.74x

What is a function?

It is defined as a special type of relationship, and they have a predefined domain and range according to the function every value in the domain is related to exactly one value in the range.

We have a function:

g(x)=19.60+1.74x

Plug x = 30

g(30) = 19.60+1.74(30)

g(30) = 19.60 + 52.2

g(30) = 71.8

Thus, the value of the g(30) is 71.8 if the function is g(x) = 19.60 + 1.74x

Learn more about the function here:

brainly.com/question/5245372

#SPJ2

please help me! no links please!

Answers

I have a link but it’s not fake

Answer:

70°

Step-by-step explanation:

hope it helps you

have a great day!!!

For the rectangular prism, the length is 3 1/2 inches, and the height is 5 1/2 inches.
What is the width?

Show your work or justify your solution.

Answers

Answer:

no answer

Step-by-step explanation:

This is because,before finding the width,length,and height of the block, there should be a stated volume,or some other parameters or properties of the prism must be stated, as such the answer cannot be determined

Answer:

thanks

Step-by-step explanation:

Pls help!! I can’t understand this

Answers

Answer: The answer is 4 units to the left of 11

Step-by-step explanation: First you get the answer -4+11=7 so then on the number line you have to find which of those options gets you 7 which will be 4 units to the left of 11 since the left is going less then 11 in the end you will get 2 places more to the right of the 5 on the number line making the number 7 there.

What is 3000×3000 4000

Answers

Answer:

I think the answer is 12,0000

Answer:

3000x3000=9,000,000

Step-by-step explanation:

3x3x1,000,000

Solve the inequality.
-7/8 < m - 13/8

Answers

Answer:

m>3/4

Step-by-step explanation:

Add -13/8 to both sides:

6/8<m

Simplify:

m>3/4

Solve for the value of x and y
5y 2x 40

Answers

Answer:

x=25 =18

Step-by-step explanation:

Right angle =90°

you are given 40, therefore 2x has to equal up to 50.

2x=50 divide both sides by 2

x=25

straight line=180°

you are given 90, therefore 5y has to equal to 90.

5y=90

divide both sides by 5

y=18

X is equal to 25
Y is equal to 18

Stuck on this question need help

Answers

yes the answer is 12,56

Answer:

YES

Step-by-step explanation:

Area: πx r^2

πx2^2

3.14x4

=12.56

so the answear is yes

NO LINKS WILL MARK BRAINLIEST

Answers

Answer:

5x3=15

6x4=24

24+15=39

Step-by-step explanation:

A is correct i think!

Answer:

39 cm

Step-by-step explanation:

8 4 pairs of shoes cost $80. What is the cost of 7 pairs of shoes?​

Answers

Answer: 140

Step-by-step explanation:

Uang Indonesia: Rp 1,155,600

Uang Amerika Serikat= $80

Uang Malaysia= $331.52

PLSSS I NEED HELP—
Which expression is equivalent to -12y + 36

Answers

Answer:

3(−4y+1)

Step-by-step explanation:

Factor −12y+3

−12y+3

=3(−4y+1)


Find the HIGHEST Common
and lowest common multple
of 36 and 48 using
prime factors

Answers

Answer:

Step-by-step explanation:

Least common multiple can be found by multiplying the highest exponent prime factors of 36 and 48. First we will calculate the prime factors of 36 and 48.

Prime Factorization of 36

Prime factors of 36 are 2, 3. Prime factorization of 36 in exponential form is:

36 = 22 × 32

Prime Factorization of 48

Prime factors of 48 are 2, 3. Prime factorization of 48 in exponential form is:

48 = 24 × 31

Now multiplying the highest exponent prime factors to calculate the LCM of 36 and 48.

LCM(36,48) = 24 × 32

LCM(36,48) = 144

List of positive integer factors of 36 that divides 36 without a remainder.

1, 2, 3, 4, 6, 9, 12, 18

Factors of 48

List of positive integer factors of 48 that divides 36 without a remainder.

1, 2, 3, 4, 6, 8, 12, 16, 24

Greatest Common Factor Number

We found the factors and prime factorization of 36 and 48. The biggest common factor number is the GCF number.

So the greatest common factor 36 and 48 is 12.

Answer:

the least is 144 the highest is 12

Step-by-step explanation:

Other Questions
Which author did not write one of the three synoptic reports on the life of Jesus Christ?MatthewMarkLukeJohn If D0 = $2.00, g (which is constant) = 6%, and P0 = $40, what is the stock's expected dividend yield for the coming year? Why according to the text was the Russian revolution a failure The number of young drivers involved in motor vehicle crashes each year has declined substantially in recent years, for teenage drivers of all ages and in crashes of all severities. The number of people injured annually in crashes involving teen drivers declined by 51% between 1994 and 2013, and the number killed each year in teen driver crashes declined by 56%. Most of the decline in injuries and virtually the entire decline in fatalities occurred between 2004 and 2013. While the reasons for the declines are not entirely clear, numerous studies have shown that strong state GDL systems have contributed substantially to reductions in injury and fatal crashes of drivers aged 1517. Research also suggests that economic factors including rising gas prices and the economic recession of 2008 resulted in substantial declines in teen driving and thus teen crashes as well.Teen Driver Crashes: 19942013,AAA Foundation for Traffic Safety What kind of information does this source provide?a description of a car crash with a teen driveran opinion on the risk of accidents related to teen driversrecent statistics on car accidents involving teenage driversan explanation of the reasons teen drivers crash cars Photosynthesis convertslight energy to chemicalenergy. Which moleculesare the end product of thistransformation of energy What is the equation of the line that passes through the point (-6,3) and has aslope of -1/6? could someone please explain who stalin was and what he did? Help ASAP! 553.168 L = mL Find the value or measure. Assume that all segments that appear to be tangent are tangent. Round answers to the nearest tenth, as needed. Explain how to write an equivalent expression using the distributive property. 2(11 + y) the answer is Sample Response: To use the distributive property, distribute multiplication by 2 to both terms inside the parentheses. The products are 2(11) and 2(y). The equivalent expression is 22 + 2y. CAN SOMEONE HELP ME PLS A store purchased jeans for $15 a pair. The store increased the price of the jeans by 65%. One month later, the jeans were on sale for 15 - off the store price. How much would a customer pay for the jeans on sale? Please help In triangle ABC, angle C is a right angle. Find the value of the trig function.Find the cos(A) if c= 15, a=9, b= 12 what is the length of ML The length of a rectangle is 7x - 3 and the width is 2(x + 3). What is the perimeter of the rectangle? Please help Im failing this classWaterways can become contaminated with mercury. Mercury is an extremely harmfulchemical. If water within the Everglades National Park became contaminated with mercury.what are some adaptations that might effect animals living there. * 5(1 Point)Enter your answer PLS HELP ME!!!!!!!!!!!!!! I DON'T HAVE A LOT OF TIME !!!!!!!! help please???????????? Vanessa throws a tennis ball in the air. The function h(t) = -15t2 + 45t + 10 represents the distance in feet, that the ball is from the ground at any time in seconds, t. What is the maximum height of the ball? Which component of weather describes the weight of the atmosphere pressing down on Earth? (2 points)Air pressureAir temperaturePrecipitationWind direction